Difference between revisions of "2002 AMC 10P Problems/Problem 22"

(Solution 1)
(Solution 1)
Line 18: Line 18:
 
We can solve this problem with an application of Legendre's formula.  
 
We can solve this problem with an application of Legendre's formula.  
  
We know that there will be an abundance of factors of <math>2</math> compared to factors of <math>5,</math> so finding the amount of factors of <math>5</math> is equivalent to finding how many factors of <math>10</math> there are. Therefore, we plug in p=5 and n=2002, then plug in p=5 and n=1001 in:
+
We know that there will be an abundance of factors of <math>2</math> compared to factors of <math>5,</math> so finding the amount of factors of <math>5</math> is equivalent to finding how many factors of <math>10</math> there are. Additionally, squaring a number will multiply the exponent of each factor by <math>2.</math> Therefore, we plug in <math>p=5</math> and <math>n=2002,</math> then plug in <math>p=5</math> and <math>n=1001</math> and multiply by <math>2</math> in:
  
 
<cmath>e_p(n!)=\sum_{i=1}^{\infty} \left\lfloor \dfrac{n}{p^i}\right\rfloor =\frac{n-S_{p}(n)}{p-1}</cmath>
 
<cmath>e_p(n!)=\sum_{i=1}^{\infty} \left\lfloor \dfrac{n}{p^i}\right\rfloor =\frac{n-S_{p}(n)}{p-1}</cmath>

Revision as of 07:38, 15 July 2024

Problem

In how many zeroes does the number $\frac{2002!}{(1001!)^2}$ end?

$\text{(A) }0 \qquad \text{(B) }1 \qquad \text{(C) }2 \qquad \text{(D) }200 \qquad \text{(E) }400$

Solution 1

We can solve this problem with an application of Legendre's formula.

We know that there will be an abundance of factors of $2$ compared to factors of $5,$ so finding the amount of factors of $5$ is equivalent to finding how many factors of $10$ there are. Additionally, squaring a number will multiply the exponent of each factor by $2.$ Therefore, we plug in $p=5$ and $n=2002,$ then plug in $p=5$ and $n=1001$ and multiply by $2$ in:

\[e_p(n!)=\sum_{i=1}^{\infty} \left\lfloor \dfrac{n}{p^i}\right\rfloor =\frac{n-S_{p}(n)}{p-1}\]

\begin{align*} e_5(2002!)=&\left\lfloor\frac{2002}{5}\right\rfloor+\left\lfloor\frac{2002}{5^2}\right\rfloor+\left\lfloor\frac {2002}{5^3}\right\rfloor+\left\lfloor\frac{2002}{5^4}\right\rfloor\\ =&400+80+16+3 \\ =&499 \end{align*}

or alternatively,

$e_5(2002!)=\frac{2002-S_5(2002)}{5-1}=\frac{2002-S_5(31002_5)]}{4}=\frac{2002-6}{4}=499.$

Similarly,

\begin{align*} e_5(2002!)=&\left\lfloor\frac{1001}{5}\right\rfloor+\left\lfloor\frac{1001}{5^2}\right\rfloor+\left\lfloor\frac {1001}{5^3}\right\rfloor+\left\lfloor\frac{1001}{5^4}\right\rfloor\\ =&200+40+8+1 \\ =&299 \end{align*}

or alternatively,

$e_5(1001!)=\frac{1001-S_5(1001)}{5-1}=\frac{1001-S_5(13001_5)]}{4}=\frac{1001-5}{4}=249.$

In any case, our answer is $499-2(249)= \boxed{\textbf{(B) } 1}.$

See also

2002 AMC 10P (ProblemsAnswer KeyResources)
Preceded by
Problem 21
Followed by
Problem 23
1 2 3 4 5 6 7 8 9 10 11 12 13 14 15 16 17 18 19 20 21 22 23 24 25
All AMC 10 Problems and Solutions

The problems on this page are copyrighted by the Mathematical Association of America's American Mathematics Competitions. AMC logo.png